What is Inequality: Definition and 1000 Discussions

In mathematics, the triangle inequality states that for any triangle, the sum of the lengths of any two sides must be greater than or equal to the length of the remaining side. This statement permits the inclusion of degenerate triangles, but some authors, especially those writing about elementary geometry, will exclude this possibility, thus leaving out the possibility of equality. If x, y, and z are the lengths of the sides of the triangle, with no side being greater than z, then the triangle inequality states that




z

x
+
y
,


{\displaystyle z\leq x+y,}
with equality only in the degenerate case of a triangle with zero area.
In Euclidean geometry and some other geometries, the triangle inequality is a theorem about distances, and it is written using vectors and vector lengths (norms):






x

+

y





x


+


y


,


{\displaystyle \|\mathbf {x} +\mathbf {y} \|\leq \|\mathbf {x} \|+\|\mathbf {y} \|,}
where the length z of the third side has been replaced by the vector sum x + y. When x and y are real numbers, they can be viewed as vectors in R1, and the triangle inequality expresses a relationship between absolute values.
In Euclidean geometry, for right triangles the triangle inequality is a consequence of the Pythagorean theorem, and for general triangles, a consequence of the law of cosines, although it may be proven without these theorems. The inequality can be viewed intuitively in either R2 or R3. The figure at the right shows three examples beginning with clear inequality (top) and approaching equality (bottom). In the Euclidean case, equality occurs only if the triangle has a 180° angle and two 0° angles, making the three vertices collinear, as shown in the bottom example. Thus, in Euclidean geometry, the shortest distance between two points is a straight line.
In spherical geometry, the shortest distance between two points is an arc of a great circle, but the triangle inequality holds provided the restriction is made that the distance between two points on a sphere is the length of a minor spherical line segment (that is, one with central angle in [0, π]) with those endpoints.The triangle inequality is a defining property of norms and measures of distance. This property must be established as a theorem for any function proposed for such purposes for each particular space: for example, spaces such as the real numbers, Euclidean spaces, the Lp spaces (p ≥ 1), and inner product spaces.

View More On Wikipedia.org
  1. B

    MHB Inequality involving Zeta Function

    Prove that for $r>2$ we have $$\frac{\zeta\left(r\right)}{\zeta\left(2r\right)}<\left(1+\frac{1}{2^{r}}\right)\frac{\left(1+3^{r}\right)^{2}}{1+3^{2r}}.$$ I've tried to write Zeta as Euler product but I haven't solve it.
  2. T

    MHB Inequality Problem To solve

    I'm working on this problem. $$2x^2 + 4x \ge x^2 - x - 6$$ I got here $$2x -x \ge -3$$ But I don't know how to go from here.
  3. S

    MHB Proving inequality using Mean Value Theorem

    Need help with this exercise been stuck on it for a while i think i get the gist of what i am supposed to do but can't seem to get it to work i am definitely missing something. I set h(x)= x-2/3 - g(x) and tried using the mean value theorem on [a,b] and then tried finding the minimum value of...
  4. A

    MHB Find Least Value Inequality for $$-1<x<0$$

    Which of the following have the least value if $$-1 < x < 0$$ $$(A) -x$$ $$(B) 1/x$$ $$(C) -1/x$$ $$(D) 1/x^2 $$ $$(E) 1/x^3$$ Mmmmmmmm... I'm not sure what to do, but I'll definitely try. We can break it up into two inequalities. $$ x > -1$$ $$0 > x$$ $$\implies -x < 1, 0 < -x$$...
  5. S

    Length-Norm inequality to root-n times Length (using 2|uv| <= |u|^2 + |v|^2)

    I've been reading "The Qualitative Theory of Ordinary Differential Equations, An Introduction" and am now stuck on an inequality I am supposed to be able to prove. I am pretty sure the inequality comes from linear algebra, I remember seeing something about it in my intro class but I let a friend...
  6. A

    Derivation of the CHSH inequality

    Bell's 1971 derivation The following is based on page 37 of Bell's Speakable and Unspeakable (Bell, 1971), the main change being to use the symbol ‘E’ instead of ‘P’ for the expected value of the quantum correlation. This avoids any implication that the quantum correlation is itself a...
  7. R

    MHB Elementary proof of generalized power mean inequality

    This is problem 20b from chapter I 4.10 of Apostol's Calculus I. The geometric mean G of n positive real numbers x_1,\ldots, x_n is defined by the formula G=(x_1x_2\ldots x_n)^{1/n}. Let p and q be integers, q<0<p. From part (a) deduce that M_q<G<M_p when x_1,x_2,\ldots, x_n are not all...
  8. T

    MHB How do I solve an inequality with a quadratic function?

    Hello, I have this inequality: $$-x^2 + 4 < 0$$ Then, I get to $$-(x-2)(x+2) < 0$$ Now, how do I solve this question from here. I understand that x = -2, or x =2 but how do I use this to solve the inequality? Thanks
  9. T

    MHB Therefore, the solution set is $\boxed{ \left[ -4, 2 \right] }$.

    Solve the Ineqality $$x^2 + 2x -8 \le 0$$I know enough to factor it like this $$(x-4) (x+2) \le 0$$ So I get 4 and -2. I just don't know how to get to the answer from here which is: $$x \ge -4\cup x\le 2$$ unless I'm misreading the answer incorrectly. Thanks
  10. P

    Prove Inequality n! > n^3 for n > 5 w/ M.I.

    Homework Statement Use M.I. to prove that n! > n^3 for n > 5 The Attempt at a Solution I already proved n! > n^2 for n>4, but this is nothing like that. This is my inductive step so far. n=k+1 (k+1)! > (k+1)^3 (k+1)! - (k+1)^3 > 0[/B] (k+1)! - (k+1)^3 = (k+1)[k! - (k+1)^2]...
  11. morrobay

    Disconnect With Inequality Realism Assumption And Bells' Lambda

    Bell, QM Ideas - Science 177 1972 :" Strictly, however. a hidden variable theory could be non-deterministic; the hidden variable could evolve randomly (possibly even discontinuously) so that their values at one instant do not specify their values at the next instant" From the locality...
  12. J

    The triangle inequality in CHSH, where is the triangle?

    http://en.wikipedia.org/wiki/CHSH_inequality#Bell.27s_1971_derivation The last step of the CHSH inequality derivation is to apply the triangle inequality. I see there are relative polarization angles, but I don't see any sides have defined length to make up a triangle. Where is the triangle?
  13. anemone

    MHB Trigonometry Inequality

    Hi MHB, When I first saw the problem (Prove that $\sin^2 x<\sin x^2$ for $0\le x\le \sqrt{\dfrac{\pi}{2}}$), I could tell that is one very good problem, but, a good problem usually indicates it is also a very difficult problem and after a few trials using calculus + trigonometry method, I...
  14. anemone

    MHB Inequality Challenge X: Prove $\ge 3l-4m+n$

    There are real numbers $l,\,m,\,n$ such that $l\ge m\ge n >0$. Prove that $\dfrac{l^2-m^2}{n}+\dfrac{n^2-m^2}{l}+\dfrac{l^2-n^2}{m}\ge 3l-4m+n$.
  15. Julio1

    MHB Proving an Inequality Involving Real Numbers

    If $a,b\in \mathbb{R}^{+}.$ Show that $a>b\implies a^{-1}<b^{-1}.$
  16. E

    Bell's Inequality Explanation for Intelligent Non-Scientist

    I have literally spent all day reading and am still very much in the dark. First off does anyone have a link to detailed blow by blow account that doesn't assume an understanding of advanced maths and physics concepts and notations but will actually address the issue in depth? Here are...
  17. Euge

    MHB Integral Inequality: Prove $\left|f\left(\frac{1}{2}\right)\right|$ Bound

    Here's my first challenge! Let $f : [0,1] \to \Bbb R$ be continuously differentiable. Show that $\displaystyle \left|f\left(\frac{1}{2}\right)\right| \le \int_0^1 |f(t)|\, dt + \frac{1}{2}\int_0^1 |f'(t)|\, dt$.
  18. M

    Solve Rational Inequality: Find Integer Roots [-2, 3]

    Homework Statement Find all integer roots that satisfy (3x + 1)/(x - 4) < 1. The Attempt at a Solution I would do this: Make it an equation and find x such that (3x + 1)/(x - 4) = 1. 3x + 1 = x - 4 2x = -5 x = -5/2 Then check if the inequality is valid for values smaller than x and for...
  19. L

    Is There a Connection Between Inequality and the Unit Disc?

    hi there, I am trying to prove the following inequality: let z\in \mathbb{D} then \left| \frac{z}{\lambda} +1-\frac{1}{\lambda}\right|<1 if and only if \lambda\geq1. The direction if \lambda>1 is pretty easy, but I am wondering about the other direction. Thanks in advance
  20. A

    Inequality - Proof that √(a^2)<√(b^2) does not imply a<b

    Hi everyone! First of all thank you for a great forum! I downloaded the app and find it ingenious! The problem stated above is from "3000 Solved Problems in Calculus". The book solves this problem simply by stating: "No. Let a=1 and b=-2". However, I am curious to know if it is possible...
  21. R

    MHB Show that equality holds in Cauchy-Schwarz inequality if and only if....

    This is from section I 4.9 of Apostol's Calculus Volume 1. The book states the Cauchy-Schwarz inequality as follows: \left(\sum_{k=1}^na_kb_k\right)^2\leq\left(\sum_{k=1}^na_k^2\right)\left(\sum_{k=1}^nb_k^2\right) Then it asks you to show that equality holds in the above if and only if there...
  22. C

    Looking for counterexample in inequality proof

    Hi guys, I have to teach inequality proofs and am looking for an opinion on something. Lets say I have to prove that a2+b2≥2ab. (a very simple example, but I just want to demonstrate the logic behind the proof that I am questioning) Now the correct response would be to start with the...
  23. D

    How to Prove a Complex Inequality with Complex Algebra

    Homework Statement Let b and a be two complex numbers. Prove that |1+ab| + |a + b| ≥ √(|a²-1||b²-1|). Homework Equations Complex algebra The Attempt at a Solution I don't know how to proceed. I posted it here to get some ideas :p
  24. Albert1

    MHB Proving Inequality: $a^{2a}b^{2b}c^{2c} > a^{b+c}b^{c+a}c^{a+b}$

    given:$a>b>c>0$ prove:$a^{2a}b^{2b}c^{2c}>a^{b+c}b^{c+a}c^{a+b}$
  25. anemone

    MHB Can you prove the inequality challenge?

    Let $x\ge \dfrac{1}{2}$ be a real number and $n$ a positive integer. Prove that $x^{2n}\ge (x-1)^{2n}+(2x-1)^n$.
  26. Greg Bernhardt

    What is the Schwarz Inequality and Its Applications?

    Definition/Summary the Schwarz inequality (also called Cauchy–Schwarz inequality and Cauchy inequality) has many applications in mathematics and physics. For vectors a,b in an inner product space over \mathbb C: \|a\|\|b\| \geq |(a,b)| For two complex numbers a,b : |a|^2|b|^2...
  27. Greg Bernhardt

    What is the AM-GM-HM Inequality and How is it Useful?

    Definition/Summary Arithmetic-Geometric-Harmonic Means Inequality is a relationship between the size of the arithmetic mean (AM), geometric mean (GM) and harmonic mean (HM) of a given set of positive real numbers. It is often very useful in analysis. Equations {\rm AM} \geq {\rm GM}...
  28. W

    Proof for the greatest integer function inequality

    Can anyone help me prove the greatest integer function inequality- n≤ x <n+1 for some x belongs to R and n is a unique integer this is how I tried to prove it- consider a set S of Real numbers which is bounded below say min(S)=inf(S)=n so n≤x by the property x<inf(S) + h we have...
  29. Albert1

    MHB Is the Inequality of Series Proven with 1 to 99 and 2 to 100?

    prove : $\dfrac {1\times 3 \times 5\times------\times 99}{2\times 4\times 6\times --\times {100}}>\dfrac {1}{10\sqrt 2}$
  30. Albert1

    MHB Prove Inequality: $18<\sum\limits_{i=2}^{99}\dfrac 1{\sqrt i} <19$

    prove : $18<1+\dfrac {1}{\sqrt 2}+\dfrac {1}{\sqrt 3}+----+\dfrac{1}{\sqrt {99}}<19$
  31. evinda

    MHB How can I show the inequality?

    Hey! (Mmm) I have to define an asymptotic upper and lower bound of the recursive relation $T(n)=3 T(\frac{n}{3}+5)+\frac{n}{2}$. Firstly,I solved the recursive relation: $T'(n)=3 T'(\frac{n}{3})+\frac{n}{2}$,using the master theorem: $$a=3 \leq 1, b=3>1, f(n)=\frac{n}{2} \text{ asymptotically...
  32. M

    Proof of Inequality Between Lower and Upper Bounds

    Convergence of Divergent Series Whose Sequence Has a Limit Homework Statement Suppose ∑a_{n} is a series with lim a_{n} = L ≠ 0. Obviously this diverges since L ≠ 0. Suppose we make the new series, ∑(a_{n} - L). My question is this: is there some sufficient condition we could put solely...
  33. R

    MHB Proving an inequality with square roots

    This is problem 13 from section I 4.7 of Apostol's Calculus Volume 1: Prove that 2(\sqrt{n+1}-\sqrt{n})<\frac{1}{\sqrt{n}}<2(\sqrt{n}-\sqrt{n-1}) if n\geq 1. Then use this to prove that 2\sqrt{m}-2<\displaystyle\sum_{n=1}^m\frac{1}{\sqrt{n}}<2\sqrt{m}-1 if m\geq 2. I have proved the first...
  34. anemone

    MHB Can Inequalities Be Proven? A Solution to a Complex Equation

    Prove that $\sqrt{x^4+7x^3+x^2+7x}+3\sqrt{3x}\ge10x-x^2$.
  35. N

    "Violation of Bell's inequality in fluid mechanics"

    I thought it might be interesting to point out this article: Title: Violation of Bell's inequality in fluid mechanics Authors: Robert Brady and Ross Anderson (Cambridge) Abstract:
  36. K

    MHB Inequality involving fractions and several variables

    What are some simplified conditions for which: $$W\bigg(A-\frac{X}{W}\bigg)^3\bigg[X-AW-\frac{AY}{N}(B+D)-\frac{AZ}{N}(C+D+E+F+G)\bigg]+\frac{X}{N}\bigg[Y(A+H)(B+D)+AZ(C+D+E+F+G)\bigg]<0$$ **WHERE:** All of the letters are positive parameters (not constants) and: $1.$ $$A,B,C,D,E,F,G,H < N...
  37. K

    MHB Finding conditions for which an inequality holds

    Hello, I do not know if this is the right place to post this question, but I believe it falls under algebra. Please redirect me if appropriate. Question: How can I show that $$P-QR^3<\frac{R^4}{C}$$ for $$C,P,Q,R > 0?$$ Thanks.
  38. Albert1

    MHB Challenging Trigonometric Inequality: Can You Prove It?

    $0<\alpha, \beta<\dfrac {\pi}{2}$ prove :$\dfrac {1}{cos^2\alpha}+\dfrac {1}{sin^2\alpha\,sin^2\beta\, cos^2\beta}\geq 9$ and corresponding $\alpha$, and $\beta$
  39. anemone

    MHB Can You Prove $6^{33}>3^{33}+4^{33}+5^{33}?

    Prove that $6^{33}>3^{33}+4^{33}+5^{33}$.
  40. I

    Dirac notation Schwarz Inequality Proof

    Homework Statement This isn't really a problem so much as me not being able to see how a proof has proceeded. I've only just today learned about Dirac notation so I'm not too good at actually working with it. The proof in the book is: |Z> = |V> - <W|V>/|W|^2|W> <Z|Z> = <V - ( <W|V>/|W|^2 ) W|...
  41. anemone

    MHB Is There a Solution to the Challenge of Inequality?

    Given that $0<k,\,l,\,m,\,n<1$ and $klmn=(1-k)(1-l)(1-m)(1-n)$, show that $(k+l+m+n)-(k+m)(l+n)\ge1$.
  42. Saitama

    MHB Find Minimum of Inequality Expression: 0<x<π/2

    I am trying to find the minimum of the following expression: $$\frac{\sin^2x+8\cos^2x+8\cos x+\sin x}{\sin x\cos x}\,\,\,,0<x<\frac{\pi}{2}$$ I know I can bash this with calculus but the expression has a nice minimum value (=17) which makes me think that it can be solved by use of some...
  43. .Scott

    CSHS Inequality - and noise

    I never heard of the CSHS Inequality until I read it in another thread. The other interesting item was this: I think an important part of that discussion is the more hits are ignored, the easier it is for local realistic theories to score over 2.00. So I just had to try. For those familiar...
  44. M

    Matrices and rank inequality exercise

    The problem statement Let ##A ∈ K^{m×n}## and ##B ∈ K^{n×r}## Prove that min##\{rg(A),rg(B)\}≥rg(AB)≥rg(A)+rg(B)−n## My attempt at a solution (1) ##AB=(AB_1|...|AB_j|...|AB_r)## (##B_j## is the ##j-th## column of ##B##), I don't know if the following statement is correct: the columns of...
  45. anemone

    MHB What are the real values of $k$ that satisfy the trigonometric inequality?

    Find all real $k$ such that $0<k<\pi$ and $\dfrac{8}{3\sin k-\sin 3k}+3\sin^2 k\le 5$.
  46. anemone

    MHB Can You Prove Inequality Challenge II?

    The cubic polynomial $x^3+mx^2+nx+k=0$ has three distinct real roots but the other polynomial $(x^2+x+2014)^3+m(x^2+x+2014)^2+n(x^2+x+2014)+k=0$ has no real roots. Show that $k+2014n+2014^2m+2014^3>\dfrac{1}{64}$.
  47. anemone

    MHB Inequality Challenge: Prove $b^3+a^3 \le 2$

    The positive real $a$ and $b$ satisfy $b^3+a^2\ge b^4+a^3$. Prove that $b^3+a^3\le 2$.
  48. evinda

    MHB Why does the inequality stand ?

    Hello! (Wasntme) I want to find the Taylor series of the function $f(x)=\log(1+x), x \in (-1,+\infty)$. We take $\xi=0, I=(-1,1)$ It is: $$f'(x)=(1+x)^{-1}, f''(x)=-1 \cdot (1+x)^{-2}, f'''(x)=2 \cdot (1+x)^{-3} , f^{(4)}(x)=-6 \cdot (1+x)^{-4}, f^{(5)}(x)=24(1+x)^{-5}$$ So,we see that...
  49. B

    MHB Maximum volume using AM GM inequality

    Hi everyone, I'm a bit confused with this question. An airline demands that all carry-on bags must have length + width + height at most 90cm. What is the maximum volume of a carry-on bag? How do you know this is the maximum? [Note: You can assume that the airline technically mean "all carry...
Back
Top